Re: Paradosso fisico (dinamica classica newtoniana)

Messaggioda Shackle » 06/11/2019, 21:20

Falco5x ha scritto:....
Se ti fidi della mia soluzione "minimal" che è questa
\[\sqrt{2g}t={{x}^{\frac{1}{4}}}\left( \frac{6}{5}x+4 \right)\]

spiega tu come mai esce questo moto senza causa
A me non vogliono credere quando invoco la singolarità della curva sull'origine come causa di ciò.


Stavo appunto cercando di capire come fai a trovare la soluzione "minimal" sopra riportata ( hai detto all'inizio con il teorema dell' Hopital) , visto che la "big one" per dt è quella cosa simpatica che dà Wolfram...La più carina di tutte , in verità , e anche la piu semplice, è quella con lo sviluppo in serie ... :-D :-D :-D

Eppure la tua minimal deve potersi ricavare dalla big one con opportune semplificazioni.. :roll:

Nel frattempo, Ho disegnato con Geogebra la funzione : $y=sqrt(x^3)$

questo è il grafico :

Testo nascosto, fai click qui per vederlo
Immagine


poi ho allargato il grafico, come consente geogebra, dettagliando l'origine :

Testo nascosto, fai click qui per vederlo
Immagine


nella seconda figura, il lato del primo quadrato misura $0.0001$ , il quadratino è $1/5$ di questo valore.

Però, la figura mi porta a dire che quello che ho detto qui è sbagliato , perchè mi sembra proprio l'opposto : la curvatura è nulla, il raggio di curvatura è infinito , nell'origine. Ma non vedo proprio niente di particolare nell’origine. Si potrebbe anche sviluppare la $y=sqrt(x^3)$ in serie (è lecito?), e derivare termine a termine, per trovare y’ e y”, oppure si possono rappresentare graficamente le funzioni derivate già trovate.
Falco, sei sicuro della tua soluzione minimale? Quale sarebbe la corrispondente funzione $y=f(x)$, che vorrei riportare sul grafico per paragonarla a quella già detta?
Ultima modifica di Shackle il 07/11/2019, 07:33, modificato 1 volta in totale.
We look for patterns when we are hungry or threatened, rather than bored. I don't think we needed to think about things when we were in standby mode in the ancient past.
Avatar utente
Shackle
Cannot live without
Cannot live without
 
Messaggio: 2509 di 8220
Iscritto il: 06/10/2016, 18:53

Re: Paradosso fisico (dinamica classica newtoniana)

Messaggioda dRic » 07/11/2019, 01:02

Falco5x ha scritto:Mi spiace ragazzi, ma non sono d'accordo con chi dice che la curva sia tracciabile.

Non voglio sembrare arrogante, ma secondo questa mi sa un po' di presa di posizione... Allora anche costruire un cerchio è impossibile perché lungo l'asse x avrei due punti a derivata infinita (tangente perpendicolare) oppure non sarei nemmeno in grado di costruire un triangolo perché la derivata di $y = |x|$ non esiste proprio in 0. Non ha senso secondo me.

Comunque, lasciamo da parte questo discorso. Credo di aver "scovato" l'errore nel tuo procedimento.

L'errore è che tu cerchi di risolvere le equazioni del moto con condizioni iniziali nulle andando a cercare le soluzioni e ipotizzando che esse abbiano la forma $y = x^k$, ma in realtà $y = x^k$ sono soluzioni delle equazioni del moto per diverse condizioni al contorno, ovvero condizioni non nulle. Mi spiego meglio.

$y = x^k$ è già una soluzioni delle equazioni di Newton. E' una soluzione implicita perché assume che, una volta calcolate $y = y(t)$ e $x = x(t)$ tu le abbia combinate eliminando la dipendenza dal tempo (un po' come quando risolvi il moto di un proiettile). Ma per risolvere un set di equazioni differenziali in $x(t)$ e $y(t)$ hai bisogno di condizioni al contorno! Quindi nel momento che tu sfrutti la relazione $y = x^k$ e la vai a sostituire dentro le eq di Newton (come hai fatto) stai implicitamente dicendo che le condizioni al contorno non sono più quelle che vuoi tu, ma sono quelle necessarie ad avere come soluzione $y = x^k$. Questo "false" condizioni al contorno prevedono appunto che l'accelerazione in zero diverga... cosa fisicamente non accettabile.

Infatti se ci pensi, l'esempio che hai proposto, nella sua stravaganza è coerentissimo: l'accelerazione in zero diverge ed infatti per mettere in moto un corpo fermo senza applicare forze avresti bisogno di una accelerazione infinita.

In conclusione, il sistema

$$ m \frac {d^2 x}{dt^2} = F_x$$
$$ m \frac {d^2 y}{dt^2} = F_x$$
$$ \text { Condizioni iniziali: } x(0) = 0, y(0) = 0, \mathbf v(0) = 0, \mathbf a(0) = 0$$
$$ \text{ vincolo: } y = x^3/2 $$

ha come soluzione unica quella nulla ovvero x = y = 0. Se invece metti delle altre condizioni al contorno avrai come soluzione alle equazioni del moto una $x(t)$ e una $y(t)$ tale che $y = x^{3/2}$, tuttavia le condizione al contorno dovranno essere diverse! (Lo dice il teorema di unicità della soluzione... i matematici lo sapranno sicuramente meglio di me)

Allego un esempio che penso ti possa chiarire le idee circa la mia spiegazione

Testo nascosto, fai click qui per vederlo
prendiamo un oscillatore armonico
$$ m \ddot x = -kx$$
e poniamo il vincolo che $v(t = 0) = 0$
adesso proviamo a risolvere andando a sostituire il $x = A*sin(\omega*t)$. Facciamo tutti i conti e ci troviamo il nostro valore di $A$. Adesso però ci accorgiamo di un paradosso! Cavolo ma $\frac d {dt} sin(t) \right]_{t = 0} = 1$... Quindi la velocità non è zero! Ma allora non posso costruire un oggetto attaccato a una molla che parta da fermo!? Deve per forza partire in movimento?

Risposta: No! Abbiamo sbagliato a cercare soluzioni del tipo $x = sin(t)$ perché sono in disaccordo con le condizioni al contorno. La scelta giusta era $x = cos(t)$ e infatti, prendendo il coseno, tutto torna.


Non ho mai scritto così tanto in questo forum, ma sono cosciente che il ragionamento è un po' arzigogolato. Spero di chiarire una volta per tutti i tuoi dubbi, altrimenti me ne tiro fuori perché non so più dove andare a parare. Nel caso non ti abbia convinto spero qualcun altro possa aiutarti a fare luce sul problema. Ciao :)
dRic
Advanced Member
Advanced Member
 
Messaggio: 1027 di 2164
Iscritto il: 01/01/2017, 03:54

Re: Paradosso fisico (dinamica classica newtoniana)

Messaggioda Falco5x » 07/11/2019, 07:49

dRic ha scritto:Non voglio sembrare arrogante, ma secondo questa mi sa un po' di presa di posizione...


Ma no dai! mi spiace che tu la pensi così, non è nel mio stile prendere posizioni preconcette! Se mi dimostrano che le ragioni sono altre sono ben contento di convincermene. Dicevo che la curva non è tracciabile perché qualunque livello di scala si assuma, la forma apparente è sempre la stessa, quella che ha disegnato anche Shackle, e non rende assolutamente conto di questa singolarità sulla derivata seconda all'origine, perché a vederla assomiglia sempre alla curva $y=x^2$, molto più innocua che non ha quella singolarità.
Ma andiamo oltre.
Io adesso non ho il tempo di ragionare bene su tutto il tuo discorso, ci penserò quando ne avrò la possibilità.
Così a prima vista i tuoi discorsi mi lasciano un po' perplesso, perché se la mia soluzione è sbagliata allora invito te a risolvere il seguente problema in modo giusto:
Problema.
Determinare la legge oraria, t(x) oppure x(t), di un corpo situato alla sommità di una guida liscia, e vincolato a non distaccarsene, avente la forma $y=x^(3/2)$, inizialmente fermo e soggetto a un campo gravitazionale verticale g.

Ecco, io credo di averlo risolto.
Tu dici che legando x(t) con y(t) faccio una forzatura delle condizioni iniziali velocità e accelerazione... ma è la forma della guida che impone queste condizioni, fa parte dei dati del problema! le condizioni iniziali in funzione del tempo sono libere, sono quelle che impongo io, a prescindere dalla forma della traiettoria!

Comunque facciamo un piano di lavoro, se sei disponibile.
Io ragiono meglio sulle tue osservazioni e tu risolvi il problema che ho scritto nel modo in cui pensi sia giusto risolverlo. Il mio procedimento sta nel primo post, e io penso che sia giusto, tanto che anche Palliit e Shackle giungono allo stesso mio integrale finale, ma tu segui pure una strada tua e vediamo cosa ti viene fuori.
Chuck Norris ha contato fino a infinito. Due volte.
Avatar utente
Falco5x
Cannot live without
Cannot live without
 
Messaggio: 2309 di 3707
Iscritto il: 08/02/2009, 19:43

Re: Paradosso fisico (dinamica classica newtoniana)

Messaggioda Shackle » 07/11/2019, 08:27

Falco5x, per chiarezza: io mi sono servito della tua soluzione, non ho affrontato il problema ex novo alla tua maniera. Per quanto concerne il moto del grave sulla guida liscia che proponi, ritorno sempre a quello che ho detto tempo fa :

$mveca =vecR +mvecg$

e si può andare avanti solo col principio di conservazione dell’energia essendo il vincolo liscio.
We look for patterns when we are hungry or threatened, rather than bored. I don't think we needed to think about things when we were in standby mode in the ancient past.
Avatar utente
Shackle
Cannot live without
Cannot live without
 
Messaggio: 2511 di 8220
Iscritto il: 06/10/2016, 18:53

Re: Paradosso fisico (dinamica classica newtoniana)

Messaggioda dRic » 07/11/2019, 08:44

Falco5x ha scritto:ma è la forma della guida che impone queste condizioni

Si ma il vincolo $y = x^k$ è rispettato se prendo $x=0$ e $y=0$. In particolare il caso $x=0$ e $y=0$ è l'unica soluzione che rispetta io vincolo E che contemporaneamente rispetta le condizioni iniziali di velocità, posizione e accelerazione nulla.

Il problema che tu provini di risolvere io lo risolvo così

Dico a priori che la soluzione è quella nulla e vado a controllare che le eq di Newton siano soddisfatte. Dunque per x(t) = y(t) = 0

- Le eq differenziali del moto hanno soluzione ? Sì! 0 è la soluzione banale.
- le condizioni al contorno su v(0), a(0), x(0) e y(0) Sono soddisfatte ? Certo!
- il vincolo della guida "y=x^k" e' soddisfatto? Si!

Allora l'unicità della soluzione mi dice che quella è l'unica soluzione al problema.

PS: tu ti stai complicando la vita chiedendomi di risolvere le equazioni differenziali per condizioni al contorno generiche, ottenendo x(t) e y(t) in funzione di parametri. Poi applicando le condizioni al contorno calcolare questi parametri. Il tutto soggetto al vincolo "y=x^k". Ma e' na roba impossibile! Non ci provo neanche a risolverlo! :D

PSS: Il tuo metodo (se è corretto, su questo non mi esprimo ancora) va benissimo per un punto qualsiasi in cui hai io corpo in un punto qualsiasi con $a(t=0) != 0$. Purtroppo è inconsistente con il caso a cui lo vuoi applicare
dRic
Advanced Member
Advanced Member
 
Messaggio: 1028 di 2164
Iscritto il: 01/01/2017, 03:54

Re: Paradosso fisico (dinamica classica newtoniana)

Messaggioda mgrau » 07/11/2019, 09:59

Ragazzi, il mio suggerimento è di spostare la discussione in un stanza di analisi, perchè, , perdonate l'arroganza, questo non è un problema di fisica.
Anche il titolo potrebbe essere cambiato, e diventare qualcosa come "Dove sta l'errore?".

In fisica, la soluzione è ovvia, ossia il corpo sta fermo. La considerazione di base, che a quanto vedo nessuno ha ritenuto di prendere in considerazione, è che non c'è motivo che cada da una parte piuttosto che dall'altra (il vecchio, ma sempre buono, principio di ragion sufficiente).
Ne segue che qualunque soluzione per cui il corpo cade è sbagliata.

La situazione me ne richiama altre, di cui presento tre esempi.
1) la trattazione classica del corpo nero: la previsione della catastrofe ultravioletta mostra che la soluzione è sbagliata.
2) gli specialisti del gioco del lotto, che a volte ti riempiono la testa di calcoli complicatissimi per dimostrare il loro sistema infallibile
3) gli scopritori del moto perpetuo
Nei tre casi il problema, per chi avesse voglia di occuparsene, sta solo nel trovare l'errore: interessante certo nel primo caso, ma molto meno negli altri due.
Vogliamo pensare che questo paradosso apra uno spiraglio su un nuovo capitolo nella fisica? Personalmente, per quel che vale, non lo credo.
mgrau
Cannot live without
Cannot live without
 
Messaggio: 5105 di 14089
Iscritto il: 29/11/2016, 11:10
Località: Milano

Re: Paradosso fisico (dinamica classica newtoniana)

Messaggioda Palliit » 07/11/2019, 12:07

Rimango dell'opinione che in modo subliminale ho fatto passare nel mio precedente post: l'errore, a mio avviso, sta nel trattare un problema che ha certe condizioni iniziali ben precise (vale a dire: $x(0)=0$ ) approssimando qualcosa in modo lecito per piccoli valori di $x$. Questo a mio avviso rende credibile il seguito soltanto se si è disposti ad accettare con una certa elasticità le condizioni iniziali: la particella inizia la sua avventura non esattamente in $0$ ma in un punto estremamente vicino a $0$. Questo, tuttavia, data l'instabilità della particella esattamente nell'origine, cambia radicalmente le cose.

Per curiosità ho provato ad analizzare il problema in altro modo, considerando che la componente orizzontale $ddotx$ dell'accelerazione è fornita dalla reazione normale della superficie. Salvo miei errori, si perviene all'equazione non lineare del second'ordine:

$ddotx=g*(kx^(k-1))/(1+k^2*x^(2k-2))" "$,


per la quale non vedo molte alternative al fatto di cercare soluzioni numeriche/grafiche. In particolare, nel caso $k=3/2$ dovrebbe ottenersi:

$ddotx=6g*sqrt(x)/(4+9x)" "$;


se qualcuno/a ha familiarità con programmi in grado di risolverla lo/la prego di provarci e farmi/ci sapere, io ho provato con quell'odioso Wolfram che non mi permette di scegliere le c.i., ma magari è un dispetto che fa soltanto a me perché sa che lo odio.



Quanto alle supposte (nel senso di participio passato) patologie della curva nell'origine, basta considerare l'estensione $f(x)=|x|^(3/2)$, che riproduce la curva (verde) in modo simmetrico nel semipiano di ascisse negative. In effetti nell'origine è piuttosto appuntita, diciamo che è meglio non sedercisi sopra, tuttavia l'andamento della derivata prima (blu/viola) è ben definito, con un simpatico flesso verticale nell'origine che giustifica una derivata seconda (rossa) tendente a $-oo$ per $x to 0$ :

Immagine


ma in ogni caso non mi sembra così strampalata.


Se credete che sia il caso di spostare la discussione in Analisi, come suggerisce @mgrau, non ho alcun problema a farlo, ditemi voi.

EDIT: come viene chiarito alcuni interventi più avanti, l'equazione differenziale che propongo in questo post come risolutiva del problema è errata perchè nasce da considerazioni inesatte.
Inoltre, il grafico pubblicato va inteso con l'asse delle ordinate orientato verso il basso, a dispetto dei valori numerici indicati.
Palliit
Moderatore
Moderatore
 
Messaggio: 3078 di 6780
Iscritto il: 28/02/2012, 21:56

Re: Paradosso fisico (dinamica classica newtoniana)

Messaggioda Falco5x » 07/11/2019, 12:22

dRic ha scritto:Dico a priori che la soluzione è quella nulla e vado a controllare che le eq di Newton siano soddisfatte. Dunque per x(t) = y(t) = 0

- Le eq differenziali del moto hanno soluzione ? Sì! 0 è la soluzione banale.
- le condizioni al contorno su v(0), a(0), x(0) e y(0) Sono soddisfatte ? Certo!
- il vincolo della guida "y=x^k" e' soddisfatto? Si!

Allora l'unicità della soluzione mi dice che quella è l'unica soluzione al problema.

PS: tu ti stai complicando la vita chiedendomi di risolvere le equazioni differenziali per condizioni al contorno generiche, ottenendo x(t) e y(t) in funzione di parametri. Poi applicando le condizioni al contorno calcolare questi parametri. Il tutto soggetto al vincolo "y=x^k". Ma e' na roba impossibile! Non ci provo neanche a risolverlo! :D

Approfitterò di queste tue osservazioni per cercare di spiegarti meglio che posso quale sia il punto.
In un caso "normale", cioè con una guida y(x) avente un andamento continuo fino alla derivata seconda, quello che dici tu sarebbe confortato anche dal metodo che ho proposto io, perché i due metodi porterebbero alle stesse conclusioni. Non ci piove.
E per essere precisi, tu dici che non essendoci né velocità né accelerazione nel punto iniziale il corpo DEVE restare fermo. E lo direi anch'io in quel caso. Allora, come verifica, io potrei però risolvere l'equazione differenziale del moto generale lungo quella traiettoria, mettendoci anche un punto di partenza generico e una velocità iniziale generica, se preferisci, e poi portando a zero queste condizioni iniziali scoprirei che il corpo resta fermo sulla sommità. Ebbene, capisco che ho infarcito di calcoli repulsivi il mio post iniziale, ma chi avesse la pazienza di guardarci dentro scoprirebbe che accade proprio così nel caso della curva $y=x^2$, che è continua in tutte le sue derivate.
Se fosse successo questo anche per la curva $y=x^(3/2)$ io non mi sarei nemmeno scomodato a pubblicare quel post, non ho certo bisogno di conforti per fare calcoli del genere.
Ho visto invece che nonostante la logica dica che in teoria la soluzione banale dovrebbe essere quella coerente con le condizioni iniziali, la soluzione calcolata nel tempo invece non contempla quella soluzione banale. Come mai?
Ti faccio notare che la soluzione trovata da Palliit qualche pagina addietro è interessante perché ipotizza questo caso con una differenza rispetto a me: prevede una $v_0$ generica. In quel caso converrai anche tu che il corpo non resta fermo ma comincia a scendere lungo la curva. Eppure facendo il limite per questa velocità iniziale tendente a zero, la funzione integranda di Pallit coincide con quella di Sheckle e con la mia: e salvo miei errori di integrazione, la soluzione banale non rientra tra quelle previste.
Ora io chiedo: come mai la soluzione calcolata in quest'ultimo modo collide con la logica che vorrebbe fermo il corpo sul punto (0,0) per ogni t?
Il punto è proprio questo: non c'è coerenza tra il metodo analitico che calcola x(t) per t qualsiasi e le considerazioni newtoniane inerenti le condizioni iniziali che vorrebbero il corpo fermo indefinitamente.
E' questo il motivo per cui ho pubblicato il post.
Poi io posso avere idee mie e non condivise da tutti sulle cause di ciò, ma questo è secondario.
Fatto sta che c'è qualcosa che non va, e dire che un risultato è giusto e l'altro è sbagliato quando i due metodi dovrebbero portare alle stesse conclusioni, ma invece ciò non accade, è quanto meno azzardato.
Chuck Norris ha contato fino a infinito. Due volte.
Avatar utente
Falco5x
Cannot live without
Cannot live without
 
Messaggio: 2310 di 3707
Iscritto il: 08/02/2009, 19:43

Re: Paradosso fisico (dinamica classica newtoniana)

Messaggioda Falco5x » 07/11/2019, 12:35

Palliit ha scritto:Rimango dell'opinione che in modo subliminale ho fatto passare nel mio precedente post: l'errore, a mio avviso, sta nel trattare un problema che ha certe condizioni iniziali ben precise (vale a dire: $x(0)=0$ ) approssimando qualcosa in modo lecito per piccoli valori di $x$. Questo a mio avviso rende credibile il seguito soltanto se si è disposti ad accettare con una certa elasticità le condizioni iniziali: la particella inizia la sua avventura non esattamente in $0$ ma in un punto estremamente vicino a $0$. Questo, tuttavia, data l'instabilità della particella esattamente nell'origine, cambia radicalmente le cose.


Ottimo Palliit :smt023 , interessante considerazione che condivido in pieno.
Questo tutto sommato mi sembra conforti l'ipotesi che accada tutto ciò perché la curva ha una singolarità nei pressi dell'origine che la rende "non fisica", ma solo matematica. Hai espresso forse con parole più convincenti delle mie questo concetto. La stranezza sparisce se si prendono curve con esponente k>=2, perché in quel caso si ha continuità fino almeno alla derivata seconda.
Riguardo all'idea di portare il problema ai matematici... non avrei molta fiducia perché loro non sono sensibili alla intuizione fisica e guardano solo l'aspetto matematico.
Per risolvere l'equazione che proponi ci sono tool automatici, come ad esempio quello che aveva usato Sheckle per integrare, e che dava un risultato intrattabile in modo elementare. Siccome il tuo secondo metodo dovrebbe essere equivalente al calcolo energetico, dubito che ci sia una soluzione elementare.
Come ho detto, io ho risolto semplificando l'analisi limitandomi all'immediato intorno dell'origine col metodo "ospedaliero". Tu che ne pensi della mia soluzione?
Chuck Norris ha contato fino a infinito. Due volte.
Avatar utente
Falco5x
Cannot live without
Cannot live without
 
Messaggio: 2311 di 3707
Iscritto il: 08/02/2009, 19:43

Messaggioda anonymous_0b37e9 » 07/11/2019, 12:53

Falco5x ha scritto:Prova un po' a pensare come sia possibile tracciare una curva che abbia pendenza zero nell'origine (e questo risponde alla prima tua osservazione: pendenza zero comporta accelerazione zero perché la curva è ortogonale alla direzione della gravità), ma appena ci si sposta di un infinitesimo assume una pendenza diversa da zero in modo discontinuo (altrimenti se la derivata prima fosse continua la derivata seconda, o meglio il suo limite destro sarebbe finito e non infinito). Una cosa del genere non è tracciabile, è solo una astrazione matematica!

Non è vero che, se la derivata prima di una funzione è "infinita" in un punto, allora la funzione debba necessariamente essere discontinua in quel punto. Insomma, affinché la derivata prima di una funzione sia "infinita" in un punto, è sufficiente che, in quel punto, il denominatore del rapporto incrementale sia un infinitesimo di ordine superiore rispetto al numeratore. Solo per fare un esempio, $y=sqrt(x)$.
anonymous_0b37e9
Cannot live without
Cannot live without
 
Messaggio: 1874 di 5111
Iscritto il: 17/07/2016, 11:55

PrecedenteProssimo

Torna a Fisica, Fisica Matematica, Fisica applicata, Astronomia

Chi c’è in linea

Visitano il forum: Nessuno e 1 ospite